Acute Comprehensive Final Exam

¡Supera tus tareas y exámenes ahora con Quizwiz!

d

The nurse working on a woman's cancer treatment floor performs nursing assessments on her assigned patients. It is most important for the nurse to report which of the following assessment findings? a) Immediate postoperative reports of throat tenderness b) Temperature of 99.2°F, pulse 72, respirations 18, blood pressure 130/80 c) Small amount of bloody drainage on surgical dressing 12 hours postoperatively d) Coolness and mottling of a newly constructed breast site

whole grains, vegetables

The nursing diagnosis for a patient is imbalanced nutrition less than body requirements related to the progress of Parkinson's disease and adverse effects of pharmacotherapy. What are the best food types to include in the plan of care?

b (If tingling, numbness, or pain is felt when the median nerve is percussed, then Tinel's sign is considered positive)

The provider asks a nurse to test a patient for Tinel's sign to diagnose carpal tunnel syndrome. The nurse asked the patient to: a) Stretch his fingers around a ball and squeeze with force. b) Hold his palm up while the nurse percussed over the median nerve. c) Make a fist and open his hand against resistance. d) Pronate his hand while the nurse palpated the radial nerve.

orchiectomy

The removal of one or both of the testes.

GH

Therapeutic Management: • Primary - use of supplemental GH • Secondary - requires removal of any tumors followed by GH therapy • Biosynthetic GH o Subcutaneous injection o Weekly dosage is 0.2 to 0.3 mg/kg o Divided into equal doses given daily for best growth o Given at bedtime

vulvar dystrophy

Thickening or lesions of the vulva; usually causes itching and may require biopsy to exclude malignancy.

b

Typical clinical manifestations of bacterial vaginosis (BV) include which of the following? a) Yellow vaginal discharge b) Malodorous, gray vaginal discharge c) Frothy, white vaginal discharge d) Thick, white adherent discharge

d

Upon examination, the nurse practitioner notes a backward positioning of the patient's uterus. How would the nurse document this finding? a) Anteflexion b) Anteversion c) Retroflexion d) Retroversion

fibroid tissue

Usually benign tumor arising from the muscle tissue of the uterus.

b

Veronica, a 17-year-old high school student, has a history of dysmenorrhea. During her monthly menses, she experiences incapacitating cramping and passes large clots. Veronica's primary care physician explains to her and her mother that he will initiate conservative treatment. What interventions would you expect the physician to recommend? a) Ultrasonography b) NSAID use c) Oral hypoglycemia agents d) Corticosteroid use

condylomata

Warty growths indicative of the human papilloma virus.

Atelectasis

What are the clinical manifestations of a patient who has _____, postoperatively? o Increasing dyspnea o Cough o Sputum production o Tachycardia o Tachypnea o Pleural pain o Central cyanosis

Sinusitis

What are the nursing interventions to promote sinus drainage for a patient diagnosed with acute _____? o Humidification of the air o Warm compresses to relieve pressure o Sudafed at first sign o Increase fluid intake o Raise HOB

Dumping syndrome

What are the signs and symptoms of _____ _____? o Early: Sensation of fullness Weakness Faintness Dizziness Palpitations Diaphoresis Cramping pains Diarrhea o Later: Rapid elevation of blood glucose Increased insulin secretion Reactive hypoglycemia

ACS

What are the signs and symptoms of _____? o Chest pain that occurs suddenly & continues despite rest & medication o SOB o Indigestion o Nausea o Anxiety o Cool, pale, moist skin o Increased RR & PR

Leukemia

What assessment data would the nurse include for a patient undergoing treatment for leukemia and at risk for fluid imbalance? o I&O o Daily weights o Assess for edema o Labs -Electrolytes -BUN -Creatinine -Hematocrit

MI

What does T-wave inversion indicate on an ECG?

--

What interventions would a nurse administer to minimize cardiac damage for a patient with a suspected MI? o Supplemental oxygen o Aspirin o Nitroglycerin o Morphine MONA

Infection

What is the most common cause of death for a patient diagnosed with acute leukemia and how would the nurse assess for this cause? -_____; VS's, labs, systemic assessment

CABG

What is the most commonly used vessel for graft site for a(n) _____? o Left internal mammary artery o Greater saphenous vein

--

What is the reason a patient would experience pain on exertion with angina pectoris secondary to atherosclerotic disease? o Increases myocardial oxygen demand

Platelet

What medications can cause altered _____ function? o Sulfa drugs o Methotrexate o Chemotherapy o Radiation

Bariatric

What should be included in the teaching plan for a patient with _____ surgery? o Ensure dietary restrictions o Eat smaller, but more frequent meals that contain protein and fiber; *each meal size should not exceed 1 cup*. -Eat only foods high in nutrients (peanut butter, cheese, chicken, fish, beans). -Consume fat as tolerated. -Ensure low-carb intake; Avoid candy. -Eat two protein snacks daily; animal protein may be poorly tolerated after Roux-en-Y bypass. -Eat slowly, chew thoroughly. -*Assume low-fowler's during mealtime and then remain in that position for 20-30 minutes after meal to delay stomach emptying and decrease the likelihood of dumping syndrome.* -Antispasmodics may aid in delaying emptying of the stomach. -Do not drink fluid with meals; consume 30 minutes before and 30-60 minutes after. -Drink plenty of water throughout day; avoid liquid calories such as alcohol, fruit drinks, non-diet sodas. -Take prescribed dietary supplements of vitamins and medium-chain triglycerides. -Monthly injections of B-12 and iron as prescribed. -Walk 30 minutes daily.

Epistaxis

What treatment would be used for a patient with _____ who has already tried applying pressure to the septum? o Cauterized with silver nitrate o Cotton tampon o Balloon-inflated catheter

cholecystitis

What type of pain does a patient with _____ have and how does it radiate? o Upper right abdomen pain that radiates to midsternal area or right shoulder

d

When a female patient reports profuse purulent discharge, dysuria, and bleeding, the advance practice nurse is most likely to prescribe which of the following medications to treat this condition? a) Terconazole (Terazol) cream inserted into the vagina at bedtime b) Tinidazole (Tindamax) one time for each partner c) Metronidazole (Flagyl), administered orally twice a day for 1 week d) Doxycycline (Vibramycin) for 1 week

D

When developing an educational program for a group of adolescents about sexually transmitted infections (STIs), what should the nurse inform the group about the single greatest risk factor for contracting an STI? a) The type of contraception used b) Where the patient lives c) The number of times the person has contact with a partner d) The number of sexual partners

c

When providing discharge teaching for a client with multiple sclerosis (MS), the nurse should include which instruction? a) "Avoid taking daytime naps." b) "Limit your fruit and vegetable intake." c) "Avoid hot baths and showers." d) "Restrict fluid intake to 1,500 ml/day."

B (malignancy of mammary ducts with early signs of erythema of nipple and areola)

When the female patient demonstrates thickening, scaling, and erosion of the nipple and areola, the nurse recognizes that the patient is exhibiting signs of which disease process? a) Acute mastitis b) Paget's disease c) Peau d'orange (edema) d) Fibroadenoma.

d

When the female patient reports a frothy yellow-green vaginal discharge, the nurse suspects the patient has a vaginal infection caused by which of the following? a) Gardnerella vaginalis b) Chlamydia c) Candida albicans d) Trichomonas vaginalis

c

Which client has the highest risk of ovarian cancer? a) 30-year-old woman taking hormonal contraceptives b) 40-year-old woman with three children c) 45-year-old woman who has never been pregnant d) 36-year-old woman who had her first child at age 22

D

Which client would the nurse identify as being at highest risk for the development of testicular cancer? a) A 39-year-old African American male who is HIV-negative b) A 45-year-old white male with a history of hypertension c) A 75-year-old white male with erectile dysfunction d) A 25-year-old male with a history of cryptorchidism

QRS complex

Which component of the ECG represents ventricular muscle depolarization?

QT interval

Which component of the ECG would represent abnormal ventricular depolarization?

b

Which discharge instruction should a nurse give a client who's had surgery to repair a hip fracture? a) "Don't flex your hip more than 120 degrees, don't cross your legs, and have someone help you put your shoes on." b) "Don't flex your hip more than 90 degrees, don't cross your legs, and have someone help you put your shoes on." c) "Don't flex your hip more than 60 degrees, don't cross your legs, and have someone help you put your shoes on." d) "Don't flex your hip more than 30 degrees, don't cross your legs, and have someone help you put your shoes on."

a (The client should avoid all activities that can result in dislocation of the hip; The affected leg should not cross midline or be turned inward; A pillow is used to keep the legs in abduction; The hip should not bend more than 90 degrees when seated; The head of bed should be kept at 60 degrees or less)

Which intervention should the nurse implement with the client who has undergone a hip replacement? a) Instruct the client to avoid internal rotation of the leg. b) Have the client bend forward to rise from the chair. c) Place the client in high Fowler's position for meals. d) Adduct the legs by placing a pillow between the legs.

d (vaginal mucus, which supports the growth of microorganisms. Monitoring and controlling blood sugars would be key in a client with diabetes. Tub baths and participation in a healthy sex life are not associated with vaginitis. Frequent douching should be avoided)

Which is the primary assessment finding in identifying the possible cause of repeated episodes of vaginitis in a female client? a) Healthy sex life b) Tub baths preferred over shower c) Occasional use of vinegar douches d) Type 2 diabetes mellitus

demerol (meperidine)

Which medication is contraindicated in a patient receiving selegiline (Eldepryl) for Parkinson's disease?

A

Which nursing assessment finding is most significant in determining the plan of care in a client with erectile dysfunction? a) Medication use b) Age c) Sexual history d) Undescended testicle

d

Which of the following are age-related changes affecting the male reproductive system? a) Prostate secretion increases. b) Patency increases. c) Testes become soft. d) Plasma testosterone levels decrease.

d

Which of the following are the earliest and the most common symptom of endometrial cancer? a) Symptoms of pressure on the bladder b) Loss of weight c) Pain d) Bleeding

A

Which of the following instructions regarding future sexual activity should a nurse give a patient with a vasectomy? a) Use a reliable method of contraception until the physician ensures that sperm are no longer present. b) Expect some bruising and incisional soreness after every sexual activity for the first 2 days. c) Administer a mild analgesic before any sexual activity. d) Sexual activity can resume after 3 weeks.

c

Which of the following is a chronic, degenerative, progressive disease of the central nervous system characterized by the occurrence of small patches of demyelination in the brain and spinal cord? a) Creutzfeldt-Jakob disease b) Parkinson's disease c) Multiple sclerosis d) Huntington disease

d

Which of the following is a disease in which there is a loss of motor neurons in the anterior horns of the spinal cord and motor nuclei of the lower brain stem? a) Alzheimer's disease b) Parkinson's disease (PD) c) Huntington disease d) Amyotrophic lateral sclerosis (ALS)

B

Which of the following is a major cause of relapsing urinary tract infections in men? a) Improper hygiene b) Chronic bacterial prostatitis c) Pyelonephritis d) Urinary retention

d

Which of the following is a primary prevention modality that aims at preventing the disease before it starts? a) Prophylactic mastectomy b) Radiation therapy c) Long-term surveillance d) Chemoprevention

c

Which of the following is a reason why ovarian cancer is largely considered to be a lethal cancer of the female reproductive system? a) Tumors present with nonspecific symptoms b) There is no effective screening test. c) All of the above d) Tumors are typically far advanced and inoperable by the time they are diagnosed.

c

Which of the following is a term used to describe excessive menstrual bleeding? a) Amenorrhea b) Dysmenorrhea c) Menorrhagia d) Metrorrhagia

A

Which of the following is accurate regarding sildenafil (Viagra)? a) Its side effects include headache, flushing, and dizziness. b) It can be taken twice daily for increased effect. c) There does not need to be sexual stimulation to produce an erection. d) The medication should be taken right before intercourse.

d

Which of the following is an anticholinergic agent used to control tremor and rigidity in Parkinson's disease? a) Levodopa (Larodopa) b) Amantadine (Symmetrel) c) Bromocriptine mesylate (Parlodel) d) Benztropine Mesylate (Cogentin)

a

Which of the following is considered a central nervous system (CNS) disorder? a) Multiple sclerosis b) Myasthenia gravis c) Guillain-Barré d) Bell's palsy

B (the status of the lymph nodes and tumor size)

Which of the following is one of the most important prognostic factors in breast cancer? a) Family history b) Status of lymph nodes c) Obesity d) Age of patient

a

Which of the following is the descent of the small intestine into the vaginal vault? a) Enterocele b) Uterine prolapse c) Cystocele d) Rectocele

c

Which of the following is the main cause of anemia in a patient with active uterine leiomyoma? a) Pressure of the fibroid on the pelvic veins b) Hemolysis c) Menorrhagia d) Poor intake of foods containing iron

c

Which of the following is the most common and most fatal primary malignant bone tumor? a) Osteochondroma b) Enchondroma c) Osteogenic sarcoma (osteosarcoma) d) Rhabdomyoma

b

Which of the following is the most common clinical manifestation of multiple sclerosis? a) Pain b) Fatigue c) Ataxia d) Spasticity

a

Which of the following is the most effective treatment for trichomoniasis? a) Metronidazole (Flagyl) b) Clindamycin (Cleocin) c) Miconazole (Monistat) d) Clotrimazole (Gyne-Lotrimin)

a

Which of the following is the most effective treatment for trichomoniasis? a) Tinidazole (Tindamax) b) Clotrimazole (Gyne-Lotrimin) c) Clindamycin (Cleocin) d) Miconazole (Monistat)

b (A patient in myasthenic crisis has severe muscle weakness, including the muscles needed to support respiratory effort. Myasthenic crisis can lead to respiratory failure and death if not recognized early. Administering IVIG, preparing for plasmaphersis, and ensuring adequate nutritional support are important and appropriate interventions, but maintaining adequate respiratory status or support is the priority during the crisis.)

Which of the following is the priority nursing intervention for a patient in myasthenic crisis? a) Administering intravenous immunoglobin (IVIG) per orders b) Assessing respiratory effort c) Ensuring adequate nutritional support d) Preparing for plasmapheresis

a

Which of the following manifestations is inconsistent with the diagnosis of Guillain-Barr syndrome? a) Cognitive decline b) Orthostatic hypotension c) Areflexia d) Ascending weakness

D

Which of the following may result if prostate cancer invades the urethra or bladder? a) Rectal discomfort b) Backache c) Hip pain d) Hematuria

C

Which of the following medications is the most effective agent in the treatment of Parkinson's disease (PD)? a) Bromocriptine mesylate (Parlodel) b) Benztropine (Cogentin) c) Levodopa (Larodopa) d) Amantadine (Symmetrel)

D

Which of the following patient populations have the highest risk for developing testicular cancer? a) Mexican American b) Asian American c) African American d) Caucasian American

C

Which of the following should nurses teach all men, especially those who have had cryptorchidism? a) Need to undergo a baseline and follow up lymph node biopsies. b) Need for blood tests to measure serum acid phosphatase levels. c) How to perform a testicular self-examination. d) Importance of regular monitoring of prostate-specific antigen (PSA) levels.

c

Which of the following suggests to the nurse that the client with systemic lupus erythematous is having renal involvement? a) Behavioral changes b) Chest pain c) Hypertension d) Decreased cognitive ability

c

Which of the following would a nurse include in a teaching plan for a client with benign prostatic hyperplasia who is not yet a candidate for surgery? a) Doing leg exercises at least daily b) Using appropriate coping to allay anxiety c) Maintaining optimal bladder emptying d) Performing deep breathing exercises periodically

c

Which of the following would be most important to include in a teaching plan for a client who has had a vasectomy? a) Taking a prescribed opioid analgesic for pain relief b) Applying warm compresses to the scrotum for the first 24 hours c) Using a reliable method of contraception for several weeks. d) Resuming sexual activity in 24 to 48 hours

B

Which of the following would not be included as a condition that can cause an elevated PSA level in the absence of prostate cancer? a) Acute prostatitis b) Erectile dysfunction c) BPH d) Acute urinary retention

d

Which position occurs when the uterus turns posteriorly as a whole unit? a) Retroversion b) Anteflexion c) Anteversion d) Retroflexion

b

While caring for a patient who is being treated for severe pelvic inflammatory disease (PID), which of the following nursing actions minimizes transmission of infection? a) Implementing reverse isolation precautions b) Performing hand hygiene when entering the room c) Keeping the patient in a sitting position d) Strictly adhering to the no visitation policy

dopamine

Why is levodopa the mainstay of treatment for a patient with Parkinson's disease? It provides a _____ precursor.

4.5

With Candidiasis, pH is _____ or less.

gastric cancer

____ ____ is a more common diagnosis among older adults, with the median age at diagnoses of 70 years in men and 74 years in women. Men are at higher risk overall.

dumping syndrome

_____ ____ is an unpleasant set of vasomotor and GI symptoms that occur in up to 76% of patients who have had bariatric surgery. It is thought that the rapid transit of the food bolus from the stomach into the intestines instead causes a rapid and exuberant release of metabolic peptides that are responsible for the symptoms of this syndrome.

dumping syndrome

_____ ____: vasomotor symptoms that occur 10-90 minutes after eating are pallor, perspiration, palpitations, headache, and feelings of warmth, dizziness, and even drowsiness. Patient may result in anorexia in fear of symptoms/reluctance to eat.

Aromatase

_____ inhibitors lower the level of estrogen in the body thereby interfering with the ability of hormone-sensitive tumors to use estrogen for growth.

First (primary union)

_____ intention: clean incision, early suture, hairline scar. Wounds are made aseptically with a minimum of tissue destruction that are properly closed heal with little tissue reaction by first intention. When would heal this way, granulation tissue is not visible and scar formation is minimal. Postop, many of these wounds are covered with a dry sterile dressing.

Second

_____ intention: gaping, irregular wound, granulation, epithelium grows over scar. Used in infected wounds or in wounds in which the edges have not been approximated.

ms

_____ is a chronic, degenerative, progressive disease of the CNS characterized by the occurrence of small patches of demyelination in the brain and spinal cord.

e coli

_____ is the most common causative organism of prostatitis. Most common urological diagnosis in men younger than 50.

dumping

_____ syndrome: physiologic response to rapid emptying of gastric contents into the jejunum, manifested by nausea, weakness, sweating, palpitations, syncope, and possible diarrhea; occurs in patients who have had partial gastrectomy and gastrojejunostomy.

NG

_____ tubes are contraindicated for decompression of the stomach when nausea or fullness occur as they may disrupt the surgical site and cause either hemorrhage or anastomotic leak after gastric or bariatric surgery.

Third

_____: Wound, increased granulation, late suturing with wide scar. Used for deep wounds that either have not been sutured early or break down and are resutured later, thus bringing together two opposing granulation surfaces. This results in a deeper, wider scar.

corticosteroids

______ include glucocorticoids; inhibit the inflammatory response to tissue injury and to suppress allergic manifestations. -Side Effects: development of diabetes, osteoporosis, peptic ulcers, increased protein break down resulting in muscle wasting, poor wound healing, redistribution of body fat. -Therapy is used extensively for adrenal insufficiency. -Early morning between 7 and 8 to keep with the natural secretion of cortisol. -Large dose therapy at 8 am, when the adrenal gland is the most active, produces maximal suppression of the gland. -A large 8 am dose is more physiologic because it allows the body to escape effects of the steroids from 4 pm to 6 am when serum levels are normally low, minimizing cushingoid effects.

HIB (hemophilius influenza B)

o A bacterium that causes several life-threatening illnesses in children younger than 5 years of age Meningitis Epiglottitis Septic arthritis

--

o HibTITER (HbOC) & ActHib (PRP-T) Require three doses o Pedvax-Hib & Comvax (PRP-OMP) Require two doses • Not given 5 years & older or under 6 weeks

ALL (acute lymphoblastic leukemia)

o Risk Factors: Male gender Age 2 to 5 White Downs syndrome, Shwachman syndrome, or ataxiatelangiectasia Xray exposure in utero Previous radiation-treated cancer o Determine Hx of varicella zoster immunization or disease Chickenpox infection may lead to disseminated, overwhelming infection

pneumonia

o Risk Factors: Prematurity Malnutrition Passive smoke exposure Low socioeconomic status Day care attendance Underlying cardiopulmonary, immune, or nervous system disease

rsv

o Risk Factors: Younger than 2 years Prematurity Multiple births Birth during April to September History of chronic lung disease Cyanotic or complicated congenital heart disease Immunocompromised Male gender Exposure to tobacco smoke Crowded living conditions Day care attendance School-age siblings Low socioeconomic status Lack of breastfeeding

MS

o Signs/Symptoms of _____: Fatigue Pain Spasticity Ataxia Tremor Diplopia (double vision) Blindness

rsv

o Signs/Symptoms: Clear runny nose Pharyngitis Low-grade fever Dev of cough 1 to 3 days into illness, followed by wheeze Poor feeding

ALL (acute lymphoblastic leukemia)

o Signs/Symptoms: Fever Recurrent infection Fatigue, malaise, or listlessness Pallor Unusual bleeding or bruising Abdominal pain N/V Bone pain H/A

ALL (acute lymphoblastic leukemia)

• Abnormal lymphoblasts abound in the blood-forming tissues • Lymphoblasts are fragile & immature o Lack infection-fighting capabilities of normal WBC o Growth is excessive & replace normal cells in bone marrow • Proliferating leukemic cells have massive metabolic needs o Results in fatigue, weight loss, or growth arrest & muscle wasting • Bone marrow unable to maintain normal RBC, WBC, & platelet levels: o Results in anemia, neutropenia, & thrombocytopenia • Bone marrow expands or leukemic cells infiltrate bone o Results in joint & bone pain • Lymphadenopathy • Hepatosplenomegaly • Spread to CNS o Vomiting, H/A, seizures, coma, vision alterations, cranial nerve palsies

osteosarcoma

• Arises from embryonic mesenchymal tissue that forms the bones • Most common sites are long bones

ALL (acute lymphoblastic leukemia)

• Classified according to the type of cell involved • Overall cure rate is 70% • Prognosis based on: o WBC count at diagnosis o Type of cytogenetic factors & immunophenotype o Age of diagnosis o Extent of extramedullary involvement • Relapse leads to poorer prognosis

ALL (acute lymphoblastic leukemia)

• Complications: o Infection o Hemorrhage o Poor growth o CNS problems o Bone problems o Testicular problems

osteosarcoma

• Complications: o Metastasis o Recurrence of disease within 3 years • Surgical removal of tumor is necessary • Chemo before & after surgery

muscular dystrophy

• Gene mutation results in absence of dystrophin (a protein that is critical for maintenance of muscle cells) • X-Linked Recessive o Mainly boys affected & receive gene from their mothers

hirschsprung

• Health History o Most newborns with HD do not pass meconium within first 24 to 48 hours o Risk Factors: -Family History of HD or Downs syndrome -Intestinal atresia -lack of normal opening

pneumonia

• Health History o Signs/Symptoms: Antecedent viral URI Fever Cough Increased RR Hx of lethargy, poor feeding, vomiting, or diarrhea in infants Chills, H/A, dyspnea, chest pain, abdominal pain, N/V in older kids

rsv

• Labs & Diagnostic Tests o Pulse oximetry may be decreased o CXR hyperinflation & patchy areas of atelectasis or infiltration o ABGs carbon dioxide retention & hypoxemia o Positive RSV via ELISA or IFA testing

ALL (acute lymphoblastic leukemia)

• Labs & Diagnostics o CBC: Low H&H Decreased RBC Decreased Platelet count Elevated, normal, or decreased WBC o Peripheral blood smear Reveal blasts o Bone marrow aspiration o Lumbar puncture o Liver Function Tests, BUN & creatinine o Chest radiography

pneumonia

• Labs & Diagnostics o Pulse oximetry may be decreased o CXR bilateral air trapping & perihilar infiltrates o Sputum culture o WBC elevated with bacterial pneumonia

hirschsprung

• Labs & Diagnostics: o Barium Enema - to look for narrowing of intestine o Rectal suction biopsy - to demonstrate an absence of ganglion cells (definite diagnosis)

muscular dystrophy

• Leads to generalized weakness of voluntary muscles that progresses over time • Hips, Thighs, Pelvis, & shoulders affected initially • As disease progresses - all voluntary & respiratory muscles are affected • Males rarely survive beyond early 30s • Late in learning to walk

muscular dystrophy

• May have pseudohypertrophy (enlarged appearance) of the calves • Preschool Years - fall often & clumsy • School Age - walks on toes or balls of feet; waddling gait; hard to raise arms • Boys ages 7 to 12 - lose ability to ambulate • Most have normal intelligence, but may exhibit specific LD

osteosarcoma

• Nursing Assessment: o Dull bone pain for several months eventually progressing to limp or gait changes o CT, MRI, Bone scan • Nursing Management o Post op care & teaching o Emotional & physical support

pneumonia

• Physical Exam o Inspection: Cyanosis following coughing spells Substernal, subcostal, or intercostal retractions Tachypnea Nasal flaring o Auscultation: Wheezes & rales in younger child Local or diffuse rales in older child Document diminished breath sounds o Percussion & Palpation: Dullness over consolidated area Tactile fremitus felt upon palpation may be increased

ALL (acute lymphoblastic leukemia)

• Physical Exam o Temp o Petechiae o Purpura o Unusual bruising

rsv

• Physical Exam o Inspection & Observation Air-hungry Cyanosis Respiratory distress Tachypnea Retractions Accessory muscle use Grunting Periods of apnea Cough Audible wheeze o Auscultation Adventitious breath sounds Early on wheeze More serious cases quiet, without wheeze • Due to significant hyperexpansion with very poor air exchange

hirschsprung

• Physical Exam: o Distended abdomen o Palpable stool masses o Rectal exam - may reveal no stool in rectum but may have a forceful expulsion of fecal material when finger is withdrawn

osteosarcoma

• Radical amputation o May use limb-sparing surgery where removed bone is replaced with endoprosthesis or cadaver bone

epiglottitis

• Radiograph of lateral neck o Done cautiously to not induce airway obstruction with changes in position of neck • DO NOT TRY TO VISUALIZE THROAT o REFLEX LARYNGOSPASM MAY OCCUR PRECIPITATING IMMEDIATE AIRWAY OCCLUSION

epiglottitis

• Signs & symptoms: o High fever o Overall toxic appearance o Refusal to speak or only with soft voice o Refusal to lie down Characteristic position is sitting forward with neck extended o Drooling o Anxiety or frightened appearance o Usually no cough

UTI

• Urinalysis may be positive for blood, nitrites, leukocytes, WBC, or bacteria • Urine Culture - positive for infecting organism • Renal US - may show hydronephrosis is child has structural defect • VCUG - not usually performed until the child has been treated with antibiotics for at least 48 hours; performed once the urine has regained sterility; may be positive for VUR

yeast vaginitis

• What are side effects of antibiotic therapy in patient with UTIs? -_____ _____occurs in 25% of patients treated with antimicrobial therapy that affect the normal flora.

PCOS

• What are some life modification for a patient with _____? -It is a complex endocrine condition resulting in chronic anovulation, androgen excess, and multiple ovarian cysts; type or hormone imbalance or cystic disorder that affects the ovaries. -Risk for: physical conditions, diabetes, cardiovascular disease, increased blood lipids.

--

• What are some nutritional guidelines for a patient with a oxalate renal calculi? -Maintain a dilute urine; limit intake of oxalate. -Many food contain oxalate, only certain food increase the urinary excretion of oxalate; e.g., spinach, strawberries, rhubarb, chocolate, tea, peanuts, and wheat bran.

penile

• What are some risk factors for penile cancer? -Rare in US; more common in Africa and South America. -Squamous cell carcinoma (most common - 90% of cases) -Risk Factors: lack of circumcision, poor genital hygiene, phimosos, HPV, smoking, UV light treatment of psoriasis on the penis, increasing age (older than 65), lichen sclerosis, and balanitis xerotica obliterans.

trichomonas vaginalis

• What are the clinical manifestations of _____ _____? -A vaginal discharge that is thin (sometimes frothy), yellow to yellow-green, malodorous, and very irritating. -An accompanying vulvitis may result, with vulvovaginal burning and itching. -Diagnosis is made often by microscopic detection of the motile causative organism or less frequently by culture. -Inspection with a speculum often reveals vaginal and cervical erythema with multiple small petechiae (strawberry spots). -Testing of trichomonal discharge reveals a pH greater than 4.5.

ovarian

• What are the clinical manifestations of _____ cancer? -Family history of a first-degree relative is the most significant risk factor. -Older than 40 years of age, industrialized countries, 5-10% are familial, early menarche, late menopause, and obesity may increase the risk. -Most women who develop ovarian cancer have no known risk factors, and no definitive causative factors have been determined.

cervical

• What are the risk factors for _____ cancer? -sexual activity, -multiple sexual partners, -early age (<20) at first coitus, -sex with uncicumcised men, -sexual contact with men whose partners had had cervical cancer, -early childbearing, -exposure to HPV types 16 and 18, -HIV and other causes of immunodeficiency, -smoking and exposure to secondhand smoke, -exposure to diethylstilbesterol in utero, -family hx of cervical cancer, -low socioeconomic status (may be related to early married, early childbearing), -nutritional deficiencies (folate, beta-carotene, -vitamin C levels are lower in women with cervical cancer), -chronic cervical infection, -overweight status.

--

• What are the risk factors for vaginal infections? -Premenarche, -pregnancy, -perimenopause, -menopause, -poor personal hygiene, -tight undergarments, -synthetic clothing, -frequent douching, -allergies, -use of oral contraceptives, -long-term or repeated use of broad spectrum antibiotics, -diabetes, -low estrogen levels, -intercourse with infected partner, -oral-genital contact (yeats can inhabit the mouth and intestinal tract), -HIV.

--

• What are the signs and symptoms of rejection in a patient with a kidney transplant? -Hyperacute rejection - within 24 hours; caused by an immediate antibody-mediated reaction that leads to generalized glomerular capillary thrombosis and necrosis. Requires immediate removal of the transplanted kidney. -Acute rejection - within 3-14 days; patient experiences *tenderness at the transplant site, a decrease in serum creatinine values, fever, malaise, and oliguria*. Tx with immunosuppressants. -After many years

6.8

• What causes gout? o Hyperuricemia, >___mg/dL

--

• What distinguishes a cancerous lump from a non-cancerous lump? -Lesions are typically non-tender, fixed rather than mobile, hard with irregular borders. -Often no signs or symptoms of brast cancer with the exception of an abnormal mammogram. -Complaints of diffuse breast pain, tenderness with menstruation usually indicate benign breast disease. -Advanced signs of breast cancer include skin dimpling, nipple retraction, or skin ulceration.

fibroadenoma

• What is a _____? -Firm, round, movable, benign tumors. They can occur from puberty to menopause with a peak incidence at 30 years of age. These masses are nontender, and are sometimes removed for definitive diagnosis.

--

• What is a priority nursing action for a patient with myasthenic crisis admitted into the ICU? o Respiratory failure from bulbar weakness Admin atropine sulfate Endotracheal intubation maybe Mechanical ventilation maybe ABGs

mechanical ventilation

• What is an expected intervention for a patient with Guillian-Barre syndrome who has a decline in vital capacity?

between 7-8 am

• What is the best time of day to take a corticosteroid?

OA

• What is the difference between RA and OA? o _____ does not involve inflammation or autoimmunity.

--

• What is the most effective treatment for trichomonas vaginalis (Trichomoniasis)? -Metronidazole or tinidazole (Tindamax) -Both partners receive a one time loading dose, or a smaller dose three times a day for one week. -Some patients complain of an unpleasant but transient metallic taste when taking metronidazole. -Nausea, vomiting, hot, flushed feeling may occur when this medication is taken with an alcoholic beverage. Patients are strongly advised to abstain from alcohol during treatment and for 24 hours after taking Flagyl or 72 hours after taking Tindamax.

--

• What is the normal appearance for a stoma following a urinary diversion? -A healthy stoma is pink or red. -Purple, brown, or black indicates compromised vascular supply. -Mucus mixed with the urine is normal for the first few days. -Should not be sensitive to touch, bleeding, encrusted, irritated.

mastitis

• What is the treatment for _____? -Antibiotics and local application of cold compresses. -Broad spectrum antibiotics may be prescribed for 7-10 days. -Patient should wear a snug bra and perform personal hygiene carefully. -Adequate rest and hydration are important to management.

hyperbilirubinemia

• What laboratory results would the nurse expect in a patient with cholelithiasis?

--

• What medication is given to suppress the symptoms and shorten the course of genital herpes? -acyclovir (Zovirax) -valacyclovir (Valtrex) -famiciclovir (Famvir)

biopsy

• What procedure is used to confirm a diagnosis of breast cancer?

scleroderma

• What should be included in patient teaching for a patient with _____? o Meticulous skin care o Preventing effects of Raynaud's o Avoid cold temperatures o Wear mittens in the cold o Proper fitting shoes o Smoking cessation

--

• Why is fluid consumed in a patient with pyelonephritis? -Inflammation of the renal pelvis. -Hydration helps facilitate flushing of the urinary tract and helps relieve pain and discomfort. -With chronic pyelonephritis, 3-4 L of fluid per day is encouraged to dilute the urine, decrease burning on urination, and prevent dehydration.

hydrocele

-A collection of fluid, generally in the tunica vaginalis of the testis, although it also mat collect within the spermatic cord; most common cause of scrotal swelling. -1 in 10 infants at birth has a hydrocele; usually resolves without tx within the first year of life.

--

-Acute bacterial prostatitis - type I -Chronic bacterial prostatitis - type II -Chronic prostatitis/chronic pelvic pain syndrome (CP/CPP) - type III - most common -IIIA - presence of WBCs in the semen after prostate massage -IIIB - absence of WBCs in the semen after prostate massage -Asymptomatic inflammatory prostatitis - type IV

hydrocele

-Acute hydroceles occur in adults over 40; may occur in association with inflammation, infection, epididymitis, local injury, or systemic infectious disease (mumps). -Chronic hydroceles occur related to the imbalance between fluid secretion and reabsorption in the tunica vaginalis. On exam, an easily transilluminated painless, extratesticular mass is found. Hydrocele transmits light on transillumination while a hernia does not. -US to differentiate large hydroceles from testicular tumors.

ra

-Autoimmune disease of unknown origin. -Effects more females (2:1, 4:1); may be a link between RA and sex hormones. -Autoimmune reaction originates in the synovial tissue. -Environmental factors (smoking) and genetic factors coalesce to produce inflammatory and destructive synovial fluid, starting in the more distal joints. -RA synovium breaks down collagen, causing edema, proliferation of the synovial membrane, and ultimately pannus (proliferation of newly formed synovial tissue infiltrated with inflammatory cells). -Pannus destroys cartilage and erodes bones resulting in loss of articular surfaces and joint motion. -Muscle fibers undergo degenerative changes; tendons and ligament elasticity and contractile power are lost.

PID

-Complications: ectopic pregnancy, infertility due to scar tissue build up, adhesions, bacteremia with septic shock, chronic pelvic and abdominal pain, and recurring PID. -Medical Management: Broad spectrum antibiotics such as a combination or Rocephin, azithromycin, and doxycycline. -Woman may experience constipation, menstrual difficulties. -The use of condoms is essential to prevent infection and sequelae. -Educate patient about ectopic pregnancy symptoms - pain, abnormal bleeding, delayed menses, faintness, dizziness, and shoulder pain.

phimosis

-Condition in which the foreskin is constricted so that it cannot be retracted over the glans; can occur congenitally or from inflammation and edema; occurs in uncircumcised males. -If the glans is not cleaned, secretions accumulate, causing inflammation of the glans penis (balanitis), which can later lead to adhesions and fibrosis.

PID

-Exact patho unknown; thought that organisms enter the body through the vagina, pass through the cervical canal, colonize the endocervix, and move upward into the uterus. -Risk Factors: early age at first intercourse, multiple sexual partners, frequent intercourse, intercourse without condoms, sex with a partner with an STI, and a history of STIs or previous pelvic infection.

gynecomastia

-Firm, overdeveloped breast tissue typically seen in adolescent boys; firm enlargement of glandular tissue beneath and immediately surrounding the areola of the male. -Caused by hormones secreted by the testes. -Virtually always benign, resolves spontaneously in 1-2 years. -In older men; firm, tender mass(es) underneath the areola. May be diffuse or related to certain medications (Digitalis, Zantac). -The excess tissue is surgically removed through a small incision around the areola.

BPH

-Health history focuses on urinary tract, previous surgical procedures, general health issues, family hx of prostate disease, fitness for possible surgery. -Patient voiding diary to record frequency and urine volume. -DRE often reveals a large, rubbery, non-tender prostate gland. -UA to screen for hematuria and UTI is recommended. -PSA level if patient does not have terminal disease -American Urological Association Symptom Index or International Prostate Symptom Score to assess severity of symptoms. -Record urinary flow rate and the measurement of postvoid residual urine. -Urodynamic studies, urethrocytoscopy, and US may be performed if invasive therapy is indicated. -Perform CBC. -Assess cardiac status and respiratory function because a high percentage of patients with BPH have cardiac or respiratory disorders due to their age

candidiasis

-Infection cause be candida species or yeast; also referred to as monolial vaginitis or yeast infection. -Fungal or yeast infection caused by strains of Candida. -Candida Albicans is the most common strain. -Antibiotic agents decrease bacteria, thereby altering the natural protective organisms usually found in the vagina.

candidiasis

-Infections may occur at any time, but are more common in pregnancy or with systemic conditions such as diabetes or HIV, or when patients are taking medications such as corticosteroids or oral contraceptive agents.

prostatits

-Inflammation of the prostate gland caused by infectious agents (bacteria, fungi, mycoplasm) or various other problems (urethral stricture, prostatic hyperplasia); often associated with lower urinary tract symptoms and symptoms of sexual discomfort and dysfunction. -Most common urological diagnosis in men younger than 50. -E-coli is the most common causative organ.

PID

-Inflammatory condition of the pelvic cavity, usually from an STI. -Gonorrhea and chlamydia are common causes; most cases are from more than one causative agent. -The fallopian tubes become narrowed and scarred, which increases the risk of ectopic pregnancy, infertility, recurrent pelvic pain, tubo-ovarian abcess, etc.

candidiasis

-Manifestations may be fairly uncomplicated, occurring sporadically in healthy women, or recurrent and complicated in women who have diabetes or are pregnant, immunocompromised, or obese. -Treatment goal is to eliminate symptoms.

phimosis

-Often develops in adults as a result of inflammation, edema, and constriction because of poor hygiene or underlying conditions such as diabetes. -The thickened secretions, smegma, can become encrusted with urinary salts and calcify, forming calculi in the prepuce and increasing the risk of penile carcinoma. -Tx (when secondary to inflammation) with the application of steroidal cream to the foreskin to soften and correct the narrowness, resulting in decreased constriction. -Phimosis is the most common indication for adult circumcision; rarely medically necessary to correct the condition.

PLISSIT

-One of the most commonly used and effective models used for assessment and intervention of sexual problems. -Permission -Limited Information -Specific Suggestion -Intensive Therapy

ED

-Psychogenic and organic causes. -Psychogenic - anxiety, fatigue, depression, pressure to perform sexually, negative body image, absence of desire, and privacy, as well as trust and relationship issues. -Organic - cardiovascular disease, endocrine disease (diabetes, pituitary tumors, testosterone deficiency, hyperthyroidism, hypothyroidism), cirrhosis, chronic kidney failure, GU complications (radical pelvic surgery), hematologic conditions (Hodgkin lymphoma, leukemia), neurological disorders (neuropathies, parkinsonism, spinal cord injury, multiple sclerosis), trauma to the pelvic or genital regions, alcohol, smoking, medications, and drug abuse.

TURP

-Surgical removal of the inner portion of the prostate through an endoscope inserted through the urethra; no external skin incision is made. -Can be performed with US guidance. -The treated tissue either vaporizes or becomes necrotic and sloughs. -Outpatient procedure, less postop bleeding noted than traditional surgical prostatectomy.

PID

-Symptoms: Usually begin with vaginal discharge, dyspareunia, lower abdominal pelvic pain, and tenderness that occurs after menses. Pain may increase with voiding or with defecation. Other symptoms include fever, malaise, anorexia, nausea, headache, and possible vomiting. -On pelvic examination, intense tenderness may be noted on palpation of the uterus or movement of the cervix.

candidiasis

-Symptoms: vaginal discharge that causes pruritis (itching) and subsequent irritation. The discharge may be watery or thick but usually has a white, cottage cheese-like appearance. Symptoms are usually more severe right before menstruation and may be less responsive to treatment during pregnancy. -Diagnosis is made by microscopic identification of spores and hyphae (long, branching, filamentous structure) on a glass slide prepared from a discharge specimen mixed with potassium hydroxide.

TURP

-The surgical and optical instrument is introduced directly through the urethra to the prostate, and the gland is then removed in small chips with an electrical cutting loop. -Benchmark surgical treatment for BPH.

candidiasis

-Treatments include antifungal agents such has miconazole (Monistat), nystatin (Mycostatin), clotrimazole (Gyne-Lotrimen), and terconazole (Terazol) cream. These agents are inserted into the vagina with an applicator at bedtime. There are 1-night, 3-night, and 7-night tx courses available. Oral medications (fluconazole [Diflucan]) are available in a one-pill dose. Relief should be noted within 3 days. -Seek comprehensive gynecological examination with four or more infections in one year.

a

A 12-year-old adolescent is being seen in the primary care office where you practice nursing. She has just had her first menses, and you are advising her on how to use a tampon. Your instructions include using the least absorbent tampon and to change tampons frequently, at least every 4 to 6 hours. Which of the following conditions are you most likely trying to prevent? a) Toxic shock syndrome b) Pelvic inflammatory disease c) Vaginitis d) Cervicitis

c

A 24-year-old client is being seen by the physician in the primary care group where you practice nursing. Over the past 2 months, the client has been receiving treatment for multiple ear infections and tonsillitis. She reports vaginal discharge and itching at this appointment. What would you expect to be the cause of her vaginitis? a) Infrequent douching b) Regulated diabetes c) Antibiotics d) Increased estrogen

a

A 30-year-old client whose mother died of breast cancer at age 44 and whose sister has ovarian cancer, is concerned about developing cancer. As a member of the oncology multidisciplinary team, the nurse should suggest that the client ask the physician about which topic? a) Genetic counseling b) Mammogram c) Papanicolaou (Pap) testing every 6 months d) Contacting the American Cancer Society

b

A 32-year-old client has been diagnosed with an ovarian cyst and asks the nurse, "Should I be worried about cancer?" Which response by the nurse would be most appropriate? a) "Since you don't have any symptoms, the chances of having cancer are really remote." b) "Most cysts are benign in younger women, but it's good to have it checked out." c) "Ovarian cysts are just that, cysts and not a cause for concern." d) "Ovarian cysts can be malignant but this is a rare occurrence."

d

A 32-year-old client has been diagnosed with an ovarian cyst and asks the nurse, "Should I be worried about cancer?" Which response by the nurse would be most appropriate? a) "Since you don't have any symptoms, the chances of having cancer are really remote." b) "Ovarian cysts can be malignant but this is a rare occurrence." c) "Ovarian cysts are just that, cysts and not a cause for concern." d) "Most cysts are benign in younger women, but it's good to have it checked out."

a

A 32-year-old client is concerned with the lumps that have developed in her breasts and is fearful of cancer. She reports variability in the size of the lumps. What could be causing her condition? a) Cyclical hormonal changes b) Nicotine c) Caffeine d) Progesterone

A

A 33-year-old female patient is recovering from a modified radical mastectomy with immediate reconstruction. This is the patient's fourth day postoperatively. Her vital signs are temperature 99°F, pulse 82, respirations 18, and blood pressure 152/90. She continues to complain of considerable pain, with a pain rating of 9 on a pain scale of 1 to 10, with 10 being the worst possible pain. The patient was discharged home with a prescription for Percocet and has been taking the Percocet as prescribed. Which of the following assessment finding would cause the nurse to phone the physician? a) A well-defined oval area near the incision site b) Itching and slight erythema at the incision site c) Small amount of serous drainage at the incision site d) White blood count (WBC) of 7,000 μL

scleroderma

A 40-year-old woman was diagnosed with Raynaud's phenomenon several years earlier and has sought care because of a progressive worsening of her symptoms. The patient also states that many of her skin surfaces are "stiff, like the skin is being stretched from all directions." The nurse should recognize the need for medical referral for the assessment of what health problem?

B

A 51-year-old client is being seen by an urologist for perineal pain, low back pain, fever lasting 5 days, and painful urination. The physician confirms a diagnosis of prostatitis and orders treatment. During your client education session, which of the following recommendations do you make? a) Regularly drain the gland. b) All options are correct. c) Avoid caffeine. d) Complete the prescribed antibiotic treatment.

d

A 54-year-old woman presents to her healthcare provider's office where you practice nursing. She is very concerned that she might have breast cancer, especially after caring for her sister that recently died from the disease. Included in your discussion is the primary and most common sign of breast cancer. Which of the following would meet this criterion? a) A painful mass in the breast, most often in the upper outer quadrant b) A painless mass in the breast, most often in the lower quadrant near the nipple c) A painful mass in the breast, most often in the lower quadrant near the nipple d) A painless mass in the breast, most often in the upper outer quadrant

frontal

A _____ lobe brain abscess produces seizures, hemiparesis, and frontal headache.

a

A client comes to the clinic for a routine evaluation. During the physical examination, the nurse palpates the client's breast and finds a small lump. Which of the following would lead the nurse to suspect possible breast cancer? a) The lump is irregularly shaped. b) The lump is soft. c) The lump is mobile. d) The client reports tenderness during the palpation.

d

A client comes to the clinic reporting urinary symptoms. Which statement would most likely alert the nurse to suspect benign prostatic hyperplasia (BPH)? a) "I've had some pain in my lower abdomen lately and felt a bit sick to my stomach." b) "I'm waking up at night to urinate and I've noticed some burning, too." c) "I've had a fever and noticed I've been running to the bathroom more often." d) "I've had trouble getting started when I urinate, often straining to do so."

B

A client comes to the emergency department complaining of moderate to severe testicular pain and tenderness. Both testicles are swollen and a penile discharge is present. The client is diagnosed with viral orchitis. Which of the following would the nurse include in this client's plan of care? a) Preparing the client for incision and drainage b) Aplying ice to the scrotum c) Administering prescribed antibiotics d) Keeping the scrotum in a dependent position

B

A client diagnosed with prostate cancer is to receive brachytherapy. Which of the following would the nurse include when discussing this therapy with the client? a) Use of probes inserted using ultrasound to freeze the tissue b) Use of radioactive seeds implanted into the prostate c) Surgical castration to decrease the level of circulating testosterone d) Need for daily treatments over a 7- to 8-week period

d risk for falls secondary to pathological fractures and impaired gait/mobility)

A client has Paget's disease. An appropriate nursing diagnosis for this client is: a) Delayed wound healing b) Risk for infection c) Fatigue d) Risk for falls

a

A client has had a right modified radical mastectomy and axillary lymph node dissection. The nurse is teaching the client about measures to reduce the risk of complications. The client demonstrates understanding of the instructions when she states which of the following? a) "I need to use an electric shaver when shaving my right armpit." b) "I can lift with my right arm objects that weigh as much as 15 pounds." c) "I should tell my manucurist that it is okay to trim the cuticles on my right hand." d) "Anytime I need blood drawn, they should get the sample from my right arm."

a, c, e

A client has just been diagnosed with Parkinson's disease. The nurse is teaching the client and family about dietary issues related to this diagnosis. Which of the following are risks for this client? Select all that apply. a) Dysphagia b) Fluid overload c) Choking d) Anorexia e) Constipation

c

A client has just been diagnosed with endometriosis. Which of the following would be most appropriate to provide the client with support and guidance about treatment options? a) "If it was me, I would probably choose the medication options." b) "The test results are clear and another physician would tell you the same thing." c) "It might help to include your partner in any of the discussions about options." d) "Treatment is essential, so you really need to make a decision pretty quickly."

a

A client has undergone a total abdominal hysterectomy and bilateral salpingo-oopherectomy as treatment for endometrial cancer. When providing postoperative care to this client the nurse would be alert for signs and symptoms of which of the following? a) Bladder dysfunction b) Neurotoxicity c) Clotting deficiencies d) Leukopenia

b

A client is being treated for trichomoniasis. The client has received instructions about the prescribed drug therapy. The nurse determines that the client needs additional teaching when she states which of the following? a) "I need to avoid drinking any alcohol with this drug." b) "My partner will not need any treatment." c) "I need to take the medication three times a day for a week." d) "I might notice a metallic taste in my mouth while I'm taking the drug."

b

A client is considering breast augmentation. Which of the following would the nurse recommend to the client to ensure that there are no malignancies? a) Breast biopsy b) Mammogram c) Mastopexy d) Ultrasound

a, c, d

A client is diagnosed with polycystic ovarian syndrome. Which of the following findings would the nurse expect to assess? Select all that apply. a) Sleep apnea b) Emaciated appearance c) Hirsutism d) Impaired glucose tolerance e) Menorrhagia

C (normal is less than 4.0)

A client is having prostate-specific antigen (PSA) testing done. Which result would the nurse identify as abnormal? a) 3.8 nanograms/milliliter b) 2.7 nanograms/milliliter c) 4.6 nanograms/milliliter d) 3.2 nanograms/milliliter

a (A characteristic feature of Guillain-Barré syndrome is ascending weakness, which usually begins in the legs and progresses upward to the trunk, arms, and face. Respiratory muscle weakness, evidenced by even, unlabored respirations, is a particularly dangerous effect of this disease progression because it may lead to respiratory failure and death.)

A client is hospitalized with Guillain-Barré syndrome. Which nursing assessment finding is most significant? a) Even, unlabored respirations b) Warm, dry skin c) Urine output of 40 ml/hour d) Soft, nondistended abdomen

d

A client is ordered a vaginal cream for the treatment of a yeast infection. As the nurse provides instruction, which step for inserting vaginal medications should be highlighted? a) Do not use once menses begins. b) Eat yogurt daily. c) Lie down for 1 hour after insertion. d) Wash applicator after each use.

a

A client is prescribed androgen therapy. Which of the following would the nurse explain as a possible adverse effect? a) Weight gain b) Areolar pigmentation c) Severe bleeding from the vagina d) Decreased libido

c

A client is receiving leuprolide as part of his treatment for prostate cancer. The nurse would be alert for which of the following as a possible adverse effect? a) Muscle overgrowth b) Nipple retraction c) Gynecomastia d) Reports of increased libido

D

A client is scheduled for a transurethral rescection of the prostate (TURP). Which statement demonstrates that the expected outcome of "client demonstrates understanding of the surgical procedure and aftercare" has been met? a) "The surgeon is going to remove the entire prostate gland." b) "I'll have a small incision on my lower abdomen after the procedure." c) "I'll have to stay in the hospital for about 3 to 4 days after the surgery." d) "The surgeon is going to insert a scope through my urethra to remove a portion of the gland."

d (To help confirm ALS, the physician typically orders EMG, which detects abnormal electrical activity of the involved muscles. To help establish the diagnosis of ALS, EMG must show widespread anterior horn cell dysfunction with fibrillations, positive waves, fasciculations, and chronic changes in the potentials of neurogenic motor units in multiple nerve root distribution in at least three limbs and the paraspinal muscles. Normal sensory responses must accompany these findings.)

A client is suspected of having amyotrophic lateral sclerosis (ALS). To help confirm this disorder, the nurse prepares the client for various diagnostic tests. The nurse expects the physician to order: a) quantitative spectral phonoangiography. b) Doppler ultrasonography. c) Doppler scanning. d) electromyography (EMG).

PARKINSON'S (Early in PD the client may notice a slight slowing in the ability to perform ADLs. A general feeling of stiffness may be noticed, along with mild, diffuse muscular pain. Tremor is a common early manifestation that usually occurs in one of the upper limbs)

A client tells the nurse that he is experiencing some leg stiffness when walking and slowness when performing ADLs. Occasionally he has noted slight tremors in his hands at rest. This information leads the nurse to suspect _____.

a

A client tells the nurse that she has found a painless lump in her right breast during her monthly self-examination. Which assessment finding would strongly suggest that this client's lump is cancerous? a) Nonmobile mass with irregular edges b) Eversion of the right nipple and mobile mass c) Mobile mass that is soft and easily delineated d) Nonpalpable right axillary lymph nodes

c

A client undergoes hip-pinning surgery to treat an intertrochanteric fracture of the right hip. The nurse should include which intervention in the postoperative care plan? a) Maintaining the client in semi-Fowler's position b) Turning the client from side to side every 2 hours c) Keeping a pillow between the client's legs at all times d) Performing passive range-of-motion (ROM) exercises on the client's legs once each shift

A

A client underwent a transurethral resection of the prostate gland 24 hours ago and is on continuous bladder irrigation. Which nursing intervention is appropriate? a) Use sterile technique when irrigating the catheter. b) Restrict fluids to prevent the client's bladder from becoming distended. c) Prepare to remove the catheter. d) Tell the client to try to urinate around the catheter to remove blood clots.

c

A client who is diagnosed with premenstrual syndrome is ordered a gonadotropin-releasing hormone (GnRH). Which is a priority teaching item in the use of this drug? a) Eat 6 small meals daily b) Maintain a menstrual diary c) Increase calcium and vitamin D intake d) Reduce sugar intake

B

A client who is to have breast conservation surgery is also to undergo a setinel lymph node biopsy (SLNB). Which of the following would the nurse include in the client's preoperative teaching plan? a) The client will need less emotional support because the procedure is less invasive. b) The client's urine may have a blue-green discoloration in the first 24 hours. c) The client will most likely be admitted for an overnight stay. d) The client has an increased risk for developing lymphedema.

d

A client who underwent an anterior colporrhaphy 6 hours ago has not voided. She reports some discomfort in her suprapubic area. Which of the following would the nurse expect to do? a) Administer a stool softener. b) Obtain an order for an analgesic. c) Apply ice to the area. d) Catheterize the client.

B

A client with Stage IV prostate cancer is to receive hormone therapy. The nurse would inform the client about possible adverse effects including which of the following? a) Increased libido b) Breast tenderness c) Enhanced potency d) Deepening of voice

D

A client with benign prostatic hyperplasia doesn't respond to medical treatment and is admitted to the facility for prostate gland removal. Before providing preoperative and postoperative instructions to the client, the nurse asks the surgeon which prostatectomy procedure will be done. What is the most widely used procedure for prostate gland removal? a) Retropubic prostatectomy b) Transurethral laser incision of the prostate c) Suprapubic prostatectomy d) Transurethral resection of the prostate (TURP)

D

A client with breast cancer is receiving chemotherapy as part of the treatment plan. The client develops neutropenia and is scheduled to receive pegfilgrastim (Neulasta). The nurse would expect this drug to be given at which time? a) Every 2 to 3 weeks after chemotherapy is given b) 7 to 10 days after chemotherapy administration c) Weekly during chemotherapy administration d) Once, approximately 24 hours after chemotherapy

D

A client with erectile dysfunction is prescribed sildenafil (Viagra). Which of the following would the nurse include in the teaching plan for this client? a) "Your blood glucose level might increase when you take this drug." b) "You need to take this medication throughout the day to be effective." c) "The effects of this medication usually last for up to 48 hours." d) "You may experience headache and some flushing with this drug."

a

A client with vaginitis complains of itching and burning of the perineum. Which suggestion would be most appropriate to relieve the client's symptoms? a) Take sitz baths frequently. b) Avoid yogurt with active lactobacilli cultures. c) Use skin protectants containing zinc oxide. d) Use a pure vinegar douche daily.

d (In Guillain-Barré syndrome, polyneuritis commonly causes weakness and paralysis, which may ascend to the trunk and involve the respiratory muscles)

A client with weakness and tingling in both legs is admitted to the medical-surgical unit with a tentative diagnosis of Guillain-Barré syndrome. On admission, which assessment is most important for this client? a) Evaluation of nutritional status and metabolic state b) Evaluation for signs and symptoms of increased intracranial pressure (ICP) c) Evaluation of pain and discomfort d) Lung auscultation and measurement of vital capacity and tidal volume

d

A client, who had intracavity radiation treatment for cervical cancer 1 month earlier, reports small amounts of vaginal bleeding. This finding most likely represents: a) infection secondary to a change in vaginal flora. b) recurrence of the carcinoma. c) development of a rectovaginal fistula. d) an expected effect of the radiation therapy.

d

A client, who wishes to preserve childbearing, asks the nurse to explain how taking oral contraceptives will work in the management of endometriosis. Which is the best response by the nurse? a) "Trapping blood causes less pain or discomfort for clients with endometriosis." b) "Endometriosis is usually cured with surgical menopause." c) "Contraceptives will allow blood to be diverted to the peritoneal cavity." d) "Symptoms of endometriosis are increased during normal menstrual cycle."

bartholin's cyst

A cyst in a paired Bartholin's or vestibular gland in the vulva.

gastric cancer

A diet high in smoked, salted, or pickled foods and low in fruits and vegetables may increase the risk for ____ _____.

a

A female client has just been diagnosed with condylomata acuminata (genital warts). What information is appropriate to tell this client? a) This condition puts her at a higher risk for cervical cancer; therefore, she should have a Papanicolaou (Pap) smear annually. b) The human papillomavirus (HPV), which causes condylomata acuminata, can't be transmitted during oral sex. c) The most common treatment is metronidazole (Flagyl), which should eradicate the problem within 7 to 10 days. d) The potential for transmission to her sexual partner will be eliminated if condoms are used every time she and her partner have sexual intercourse.

b

A female client is diagnosed with breast abscess. She would like to continue to breast-feed her newborn. Which of the following would be most appropriate in this situation? a) Encourage the client to include protein content in the diet. b) Assist the client to pump the breasts to remove breast milk. c) Reduce the frequency of removing and reapplying the dressings. d) Instruct the client to wear a tight-fitting bra.

d

A female patient comes to the clinic with the complaint that she is having a greenish-colored discharge from the nipple and the breast feels warm to touch. What does the nurse suspect these symptoms may indicate? a) Blocked lymph duct b) A ruptured cyst c) Cancer d) Infection

a (greater than 10.2 is hypercalcemia)

A home care nurse assesses for disease complications in a client with bone cancer. Which laboratory value may indicate the presence of a disease complication? a) Calcium level of 11.6 mg/dl b) Sodium level of 110 mEq/L c) Potassium level of 6.3 mEq/L d) Magnesium level of 0.9 mg/dl

dopamine

A lack of which substance, resulting in an imbalance between it and acetylcholine, is the basic origin of Parkinson's disease?

c (normal is 30 mL/hr)

A nurse assesses a client shortly after living donor kidney transplant surgery. Which postoperative finding must the nurse report to the physician immediately? a) Temperature of 99.2° F (37.3° C) b) Serum sodium level of 135 mEq/L c) Urine output of 20 ml/hour d) Serum potassium level of 4.9 mEq/L

b

A nurse is assessing a client with Parkinson's disease. Which of the following would the nurse expect to find? a) Continuous tremors b) Slowing of activity c) Muscle flaccidity d) Gait with the body leaning backward

b

A nurse is assisting with a neurological examination of a client who reports a headache in the occipital area and shows signs of ataxia and nystagmus. Which of the following conditions is the most likely reason for the client's problems? a) Temporal lobe abscess b) Cerebellar abscess c) Frontal lobe abscess d) Wernicke's abscess

a

A nurse is caring for a client who underwent a subtotal gastrectomy. To manage dumping syndrome, the nurse should advise the client to: a) drink liquids only between meals. b) don't drink liquids 2 hours before meals. c) drink liquids only with meals. d) restrict fluid intake to 1 qt (1,000 ml)/day.

b

A nurse is caring for a client with acute pyelonephritis. Which nursing intervention is the most important? a) Using an indwelling urinary catheter to measure urine output accurately b) Increasing fluid intake to 3 L/day c) Administering a sitz bath twice per day d) Encouraging the client to drink cranberry juice to acidify the urine

a, c, e

A nurse is preparing a presentation for a local senior citizen group about age-related changes in the body systems. Which of the following would the nurse include as associated with the male reproductive system? Select all that apply. a) Loss of testicular firmness b) Enlargement of the testes c) Thickening of the seminiferous tubules d) Shrinkage of the prostate gland e) Decrease in sexual energy level

b

A nurse is preparing a presentation for a local women's group about benign and malignant breast conditions. Which of the following would the nurse include as an example of a malignant breast condition? a) Atypical hyperplasia b) Ductal carcinoma in situ c) Lobular carcinoma in situ d) Fibroadenoma

d

A nurse is providing care for a client recovering from gastric bypass surgery. During assessment, the client exhibits pallor, perspiration, palpitations, headache, and feelings of warmth, dizziness, and drowsiness. The client reports eating 90 minutes ago. The nurse suspects: a) Dehiscence of the surgical wound b) A normal reaction to surgery c) Peritonitis d) Vasomotor symptoms associated with dumping syndrome

b

A nurse is reviewing a client's history for possible risk factors associated with breast cancer. Which of the following would the nurse identify as increasing the client's risk? a) Menopause at age 50 years b) First full-term pregnancy at age 34 years c) One of three living children born prematurely d) Menarche at age 14 years

b

A nurse is reviewing a client's medical history. Which factor indicates the client is at risk for candidiasis? a) Use of spermicidal jelly b) Use of corticosteroids c) Menopause d) Nulliparity

B

A nurse is teaching a male client to perform monthly testicular self-examinations. Which point is appropriate to make? a) Testicular cancer is very difficult to diagnose. b) Testicular cancer is a highly curable type of cancer. c) Testicular cancer is the number one cause of cancer deaths in males. d) Testicular cancer is more common in older men.

b

A nurse receives her client care assignment. Following the report, she should give priority assessment to the client: a) who has a sodium level of 135 mEq/L and a potassium level of 3.7 mEq/L 7 days after a kidney transplant. b) who, following a kidney transplant, has returned from hemodialysis with a sodium level of 110 mEq/L and a potassium level of 2.0 mEq/L. c) with pinkish mucus discharge in the appliance bag 2 days after an ileal conduit. d) who is experiencing mild pain from urolithiasis.

a

A nurse who works in a gynecologist's office frequently cares for patients who are diagnosed with vulvovaginal candidiasis. The nurse should teach the patients how to manage and treat the most common symptom of: a) Vulvar pruritus. b) Dyspareunia. c) Vaginal pain. d) Dysuria.

c

A nurse who works in an oncology practice prepares patients for the side effects of adjuvant hormonal therapy to treat breast cancer. Which of the following is the hormonal agent that has an increased risk of pulmonary embolism and deep vein thrombosis? a) Exemestane b) Anastrozole c) Tamoxifen d) Letrozole

a (A frontal lobe brain abscess produces seizures, hemiparesis, and frontal headache)

A patient has been diagnosed with a frontal lobe brain abscess. Which of the following nursing interventions is appropriate? a) Initiate seizure precautions. b) Ensure that patient takes nothing by mouth (NPO). c) Assess for facial weakness. d) Assess visual acuity.

b

A patient has been diagnosed with a vaginal infection and received a prescription for metronidazole (Flagyl). The nurse knows that this is the recommended treatment for a vaginal infection caused by what organism? a) Streptococcus b) Trichomonas vaginalis c) Candida albicans d) Escherichia coli

a

A patient has been newly diagnosed with breast cancer. During her preoperative instructions the physician indicated that removal of the breast tissue and axillary lymph node dissection leaving the muscular structure intact was indicated as surgical treatment for her breast cancer. To which of the following procedures is the physician asking the patient to consent? a) Modified radical mastectomy b) Segmental mastectomy c) Radical mastectomy d) Total mastectomy

C

A patient has been newly diagnosed with breast cancer. During her preoperative instructions the physician indicated that removal of the breast tissue and axillary lymph node dissection leaving the muscular structure intact was indicated as surgical treatment for her breast cancer. To which of the following procedures is the physician asking the patient to consent? a) Total mastectomy b) Segmental mastectomy c) Modified radical mastectomy d) Radical mastectomy

A

A patient has been scheduled to undergo a transurethral resection of the prostate (TURP) for benign prostatic hyperplasia (BPH). The nurse knows that the patient understands preoperative teaching when he makes which of the following statements. a) "I understand I may develop urethral strictures as a result of having the TURP." b) "I will have my wife come with me to drive me home following the outpatient procedure." c) "I understand there is no danger of retrograde ejaculation following the TURP." d) "I'm worried my wife will leave me after the TURP because of the side effect of erectile dysfunction."

d

A patient has had a total mastectomy 12 hours ago and the nurse is assessing the surgical wound. The nurse observes ecchymosis, swelling, and tightness around the wound, and the patient states that it is painful. What does the nurse suspect has occurred? a) The patient has developed lymphedema. b) The patient has developed a cyst. c) The patient has developed an infection. d) The patient has developed a hematoma.

GI

A patient has just been told by his physician that he has scleroderma. The physician tells the patient that he is going to order some tests to assess for systemic involvement. The nurse knows that priority systems to be assessed include what?

b

A patient is admitted to the health care center with abdominal pain, nausea, and vomiting. She has a body mass index (BMI) of 30 and reports irregular menstrual cycles and feeling tired all the time. The nurse suspects the patient's symptoms to be which of the following conditions? a) Bacterial vaginosis b) Polycystic ovary syndrome (PCOS) c) Endometriosis d) Cancer of the cervix

c

A patient is diagnosed with amyotrophic lateral sclerosis, also known as ALS or Lou Gehrig's disease. The nurse understands that the symptoms of the disease will begin in what way? a) Ascending paralysis b) Numbness and tingling in the lower extremities c) Weakness starting in the muscles supplied by the cranial nerves d) Jerky, uncontrolled movements in the extremities

a

A patient is diagnosed with the most common type of uterine fibroid, an intramural fibroid. The nurse includes which of the following information in teaching the patient about this type of fibroid? a) It grows within the wall of the uterine muscle. b) It arises from inside or outside the surface of the uterine muscle. c) It grows below the inner uterine surface. d) It lies underneath the outermost layer of the uterus.

d

A patient is exhibiting bradykinesia, rigidity, and tremors related to Parkinson's disease. The nurse understands that these symptoms are directly related to what decreased neurotransmitter level? a) Serotonin b) Phenylalanine c) Acetylcholine d) Dopamine

b

A patient is having a DRE in the physician's office and the nurse is to assist in the examination. What can the nurse instruct the client to do to decrease the discomfort from the exam? a) Take a deep breath and hold it when the physician inserts a gloved finger into the rectum. b) Take a deep breath and exhale when the physician inserts a gloved finger into the rectum. c) When bending over the examining table, point the feet outward to decrease the discomfort. d) Inform the patient that the examination is not uncomfortable and will be over in a short period of time.

C

A patient is having a biopsy that will remove the entire mass, plus a margin of surrounding tissue. What type of biopsy will be documented on the operative permit? a) Incisional biopsy b) Ultrasound-guided core biopsy c) Excisional biopsy d) Core biopsy

b

A patient is planning to use a negative-pressure (vacuum) device to maintain and sustain an erection. What should the nurse caution the patient about with the use of this device? a) Watch for erosion of the prosthesis through the skin. b) Do not leave the constricting band in place for longer than 1 hour to avoid penile injury. c) Do not use the device while taking nitrates. d) Watch for the development of infection.

b

A patient is receiving chemotherapy with paclitaxel as treatment for ovarian cancer. The patient arrives at the facility for laboratory testing prior to her next dose of chemotherapy. The results are as follows: Hemoglobin: 12.9 gm/dL White blood cell count: 2,200 /cu mm Platelets: 250,000 /cu mm Red blood cell count: 4,400,00/cu mm Which result would be a cause for concern? a) Platelet count b) White blood cell count c) Red blood cell count d) Hemglobin level

A

A patient is suspected to have prostate cancer related to observed clinical symptoms. What definitive test can the nurse assist with to confirm a diagnosis of prostate cancer? a) Prostate biopsy b) Cystoscopy c) DRE d) PSA

urine

A patient is taking entacapone as part of therapy for Parkinson's disease. Which intervention by the nurse is appropriate at this time? Notify the patient that this drug causes discoloration of the _____.

C

A patient is to undergo a TURP for BPH. Which of the following is accurate with regard to a TURP? a) It is done on an outpatient basis. b) There is no danger of retrograde ejaculation. c) Urethral strictures are more frequent for TURP than with nontransurethral procedures. d) A TURP causes erectile dysfunction.

A

A patient is undergoing treatment for prostate cancer. He has chemotherapy sessions regularly. However, of late he is showing symptoms of food allergy and loss of appetite. He has lost considerable weight due to this. Which of the following is an appropriate nursing task in this situation? a) The nurse should ask the patient to keep a diet diary. b) The nurse must administer serotonin blockers. c) The nurse should ensure greater fluid intake. d) The nurse should include fruits in the patient's diet.

a

A patient reports to the nurse that she has a sense of pelvic pressure and urinary problems such as incontinence, frequency, and urgency. The problem has gotten much worse since the birth of her third child. What does the nurse suspect the patient is experiencing? a) A cystocele b) A rectocele c) A urinary tract infection d) An enterocele

d

A patient suspected of having Guillain-Barré syndrome has had a lumbar puncture for cerebrospinal fluid (CSF) evaluation. When reviewing the laboratory results, what does the nurse find that is diagnostic for this disease? a) White blood cells in the CSF b) Glucose in the CSF c) Red blood cells present in the CSF d) Elevated protein levels in the CSF

a

A patient undergoing treatment for vaginitis is also counseled about measures to prevent its recurrence. Which patient statement best indicates effective counseling? a) "My sexual partner will also need to be treated." b) "I will insert a vaginal suppository after intercourse." c) "I will always douche immediately after intercourse." d) "I will void immediately after intercourse."

c

A patient who is diagnosed as having endometriosis asks for an explanation of the disease. What is the nurse's best response? a) The lining of the uterus is too thin because endometrial tissue has implanted outside the uterus. b) The lining of the uterus is thicker than usual, causing heavy bleeding and cramping. c) Tissue similar to the lining of the uterus has implanted in areas outside the uterus. d) She has developed an infection in the lining of her uterus.

syncope, dizziness

A patient will be taking selegiline, 10 mg daily, in addition to dopamine replacement therapy for Parkinson's disease. The nurse will implement which precautions regarding selegiline? Monitor for _____ and _____.

c

A patient with HIV has recently completed a 7-day regimen of use of antibiotics. She reports vaginal itching and irritation. In addition, the patient has a white, cottage cheese-like vaginal discharge. Which of the following is the patient most likely suffering? a) Trichomonas vaginalis b) Human papillomavirus (HPV) c) Vulvovaginal candidiasis d) Bacterial vaginosis

GI, GU, cardio

A patient with Parkinson's disease needs an anticholinergic agent to help relieve dyskinesias. Which organ systems are priority nursing assessments before the administration of an anticholinergic agent to this patient?

on-off

A patient with Parkinson's disease will start taking entacapone along with the carbidopa-levodopa therapy he has been taking for a few years. The nurse recognizes that the advantage of taking entacapone is that The entacapone can reduce the ____-____ effects.

d

A patient with fungal encephalitis is receiving amphotericin B complains of fever, chills, and body aches. The nurse knows that these symptoms a) are primarily associated with infection with Coccidioides immitis and Aspergillus. b) indicate renal toxicity and a worsening of the patient's condition. c) indicate the need for immediate blood and cerebral spinal fluid (CSF) cultures. d) may be controlled by the administration of diphenhydramine (Benadryl) and acetaminophen (Tylenol) approximately 30 minutes prior to administration of the amphotericin.

a

A perimenopausal woman informs the nurse that she is having irregular vaginal bleeding. What should the nurse encourage the patient to do? a) See her gynecologist as soon as possible. b) Disregard this phenomenon because it is common during this life stage. c) Mention it to her physician during her next annual examination. d) Stop taking her Premarin (hormonal therapy).

enterocele

A protrusion of the intestinal wall into the vagina.

C

A triple-lumen indwelling urinary catheter is inserted for continuous bladder irrigation following a transurethral resection of the prostate. In addition to balloon inflation, the functions of the three lumens include: a) intermittent inflow and continuous outflow of irrigation solution. b) continuous inflow and intermittent outflow of irrigation solution. c) continuous inflow and outflow of irrigation solution. d) intermittent flow of irrigation solution and prevention of hemorrhage.

b

A woman in her late thirties has been having unusually heavy menstrual periods combined with occasional urine and stool leakage over the last few weeks. On further enquiry, she reveals that she also has postcoital pain and bleeding. To which of the following diagnoses do you think the investigations are most likely to lead? a) Cancer of the urinary tract b) Cervical cancer c) Colorectal cancer d) Hodgkin's disease

b

A young client presenting at the health clinic with fever and mucopurulent vaginal drainage is diagnosed with acute pelvic inflammatory disease (PID). Which long-term affect is of greatest concern in the care of this client? a) Ascending infection b) Infertility c) Chronic pelvic inflammatory disease d) Pelvic pain

fistula

Abnormal opening between two organs or sites (e.g. vesicovaginal, between bladder and vagina; rectovaginal, between rectum and vagina)

orchitis

Acute inflammation of the testes (testicular congestion) caused by pyogenic, viral, spirochetal, parasitic, traumatic, chemical, or unknown factors.

dumping syndrome

After _____ _____ symptoms are resolved with evacuation of the intestines (i.e. defecation), there is a rapid elevation of blood glucose, followed by an increased insulin secretion. This results in a reactive hypoglycemia, which also is unpleasant for the patient.

c

After a radical prostatectomy for prostate cancer, a client has an indwelling catheter removed. The client then begins to have periods of incontinence. During the postoperative period, which intervention should be implemented first? a) Self-catheterization b) Fluid restriction c) Kegel exercises d) Artificial sphincter use

600-800

After gastric/bariatric surgery: when bowel sounds return to normal and oral intake is resumed, six small feedings consisting of a total of ___-___ calories per day are provided, and consumption of fluids between meals is encouraged to prevent dehydration. Eat slowly, stop when full feeling occurs.

B

After having transurethral resection of the prostate (TURP), a client returns to the unit with a three-way indwelling urinary catheter and continuous closed bladder irrigation. Which finding suggests that the client's catheter is occluded? a) About 1,000 ml of irrigant have been instilled; 1,200 ml of drainage have been returned. b) The client reports bladder spasms and the urge to void. c) The normal saline irrigant is infusing at a rate of 50 drops/minute. d) The urine in the drainage bag appears red to pink.

d

All of the following are guidelines for avoiding hip dislocation after replacement surgery. Select the answer that is not. a) Never cross the legs when seated. b) Keep the knees apart at all times. c) Put a pillow between the legs when sleeping. d) You may flex at the hip to put on clothing such as pants, stockings, socks, or shoes.

d

Amy Patterson, a 50-year-old teacher, is visiting the OB-GYN practice where you practice nursing with complaints associated with menopause. After discussing treatment options with her healthcare provider, she meets with you to further discuss her treatment concerns. Which of the following is a likely recommendation to increase her interest in sexual activity? a) Bisphosphates b) Antidepressants c) Herbal aphrodisiacs d) Low-dose androgens

C

An 80-year-old client is being seen by the urologist because of erectile dysfunction. This client has a history of hypertension and heart disease. Which of the following would be the most likely cause of his erectile dysfunction? a) Hypertension medication b) Depression c) All options are correct. d) Atherosclerosis

varicocele

An abnormal dilation of the veins of the pampiniform venous plexus in the scrotum.

c

An elderly client is being evaluated for suspected pyelonephritis and is ordered kidney, ureter, and bladder (KUB) x-ray. The nurse understands the significance of this order is related to which rationale? a) Reveals causative microorganisms b) If risk for chronic pyelonephritis is likely c) Detects calculi, cysts, or tumors d) Shows damage to the kidneys

c

An elderly client, who can void only while standing and pushing upward on the vagina, is ordered a pessary. Which comment from the client indicates a need for further teaching about this device? a) "I will report any signs of irritation or bleeding." b) "I should have greater ease of emptying my bladder." c) "I will remove and clean it every day." d) "I should report any foul odor or drainage."

MS

An immune mediated, progressive demyelinating disease of CNS;

a

An instructor is preparing a class on renal cancer for a group of students. Which of the following would the instructor include as a possible risk factor? a) Obesity b) Exposure to sunlight c) Age below 40 years d) Female gender

priapism

An uncontrolled, persistent erection of the penis from either neural or vascular causes, including medications, sickle cell thrombosis, leukemic cell infiltration, spinal cord tumors, and tumor invasion of the penis or its vessels.

a

Assessment of a client reveals evidence of a cystocele. The nurse interprets this as which of the following? a) Bulging of the bladder into the vagina b) Herniation of the rectum into the vagina c) Protrusion of intestinal wall into the vagina d) Downward displacement of the cervix

c

Assessment of a client reveals evidence of a cystocele. The nurse interprets this as which of the following? a) Protrusion of intestinal wall into the vagina b) Herniation of the rectum into the vagina c) Bulging of the bladder into the vagina d) Downward displacement of the cervix

4

Atypical hyperplasia increases a women's risk for breast cancer about how many time compared with that of the general population?

D

Based on her knowledge of the most common site of metastatic spread for breast cancer, the nurse would recommend periodic assessments of the: a) Brain b) Liver c) Lungs d) Bone

lichen sclerosus

Benign disorder of the vulva. A chronic condition that causes thin, white patches of skin, usually in the genital or anal area. Cannot be cured but treatment may help.

peyronies disease

Buildup of fibrous plaques in the sheath of the corpus cavernosum, causing curvature of the penis when it is erect.

rectocele

Bulging of the rectum into the vagina.

liposomal therapy

Chemotherapy delivered in a liposome, a nontoxic drug therapy.

d

Clinical manifestations related to metastasis from prostate cancer include a) thrombocytopenia. b) polyuria. c) weight gain. d) hip pain.

gastric (bariatric)

Common dietary deficiencies after ____ surgery: -malabsorption of organic iron --> may require supplements of oral or parenteral iron -low serum b12 --> may be prescribed monthly b12 injections to prevent pernicious anemia

PCOS (polycystic ovary syndrome)

Complex endocrine condition resulting in chronic anovulation, androgen excess, and multiple ovarian cysts.

gastric

Complications of _____ surgery include hemorrhage, nutritional deficiencies, bile reflux, dumping syndrome, and dysphagia.

b

Culture of client's vaginal discharge reveals Gardnerella vaginalis. Which of the following would the nurse expect to assess? a) Yellowish white discharge b) Fishy smelling watery discharge c) Foul foamy discharge d) Thick curdy white discharge

brachytherapy

Delivery of radiation therapy through internal implants called seeds to a localized area of tissue.

cryotherapy

Destruction of tissue by freezing (e.g. with liquid nitrogen).

cystocele

Displacement of the bladder downward into the vagina.

c

During a breast examination, which finding most strongly suggests that a client has breast cancer? a) Bloody discharge from the nipple b) Slight asymmetry of the breasts c) A fixed nodular mass with dimpling of the overlying skin d) Multiple firm, round, freely movable masses that change with the menstrual cycle

a

During consultations with the oncologist, a patient with cervical cancer is informed that radioactive materials will be inserted in the area of her tumor. She has to undergo these sessions over a period of 3 weeks. Which of the following methods of cancer treatment is being described here? a) Brachytherapy b) Needle aspiration biopsy c) Chemotherapy d) Antineoplastic therapy

retrograde ejaculation

During ejaculation, semen travels to the urinary bladder instead of exiting through the penis.

dumping syndrome

Early symptoms of ____ ____ in the post-bariatric surgery patient: -sensation of fullness -weakness -faintness -dizziness -palpitations -diaphoresis -cramping pains -diarrhea Symptoms resolve once the intestine has been evacuated (i.e. with defecation).

endometriosis

Endometrial tissue in abnormal locations; causes pain with menstruation, scarring, and possible infertility.

temporal

Facial weakness and visual disturbances are associated with a _____ lobe abscess.

gastric cancer

Factors such as diet, male gender, chronic inflammation of the stomach, H. pylori infection, pernicious anemia, smoking, achlorhydria, gastric ulcers, previous subtotal gastrectomy (more than 20 years ago), and genetics increase the risk for _____ _____.

fibroadenoma

Firm, round, movable, benign tumors. They can occur from puberty to menopause with a peak incidence at 30 years of age. These masses are nontender, and are sometimes removed for definitive diagnosis.

b

Following a colposcopy, the confirmation of in situ carcinoma of the cervix has been determined. Which comment by the client indicates an appropriate understanding of the diagnosis? a) "I will need surgery and chemotherapy to increase my odds for survival." b) "The cancer has not spread." c) "I can wait until I have finished having babies to seek treatment." d) "I will not need any further treatment."

c

Following a radical vulvectomy, the nurse is preparing the client for discharge to home. Which care intervention would be considered the priority for this client? a) Alterations for sexual function b) Care of colostomy site c) Prevention of wound complications d) Relieving edema to lower extremities

a

Following morning hygiene of an elderly client, the nurse is unable to replace the retracted foreskin of the penis. Which is the most likely outcome? a) Painful swelling b) Erection of the penis c) Unclean glans d) Nausea and vomiting

surgery

Gastric _____ may be performed on patients with peptic ulcers wwhohave life-threatening hemorrhage, obstruction, perforation, or penetration or whose condition does not respond to meds. It may also be indicated for patients with gastric cancer or trauma. Surgical procedures include a vagotomy, pyloroplasty, a partial gastrectomy, or a total gastrectomy.

b (Myelin is a complex substance that covers nerves, providing insulation and speeding the conduction of impulses from the cell body to the dendrites. The axon carries the message to the next nerve cell. The neuron is the building block of the nervous system. A neurotransmitter is a chemical messenger.)

Guillain-Barré syndrome is an autoimmune attack on the peripheral myelin sheath. Which of the following is an action of myelin? a) Acts as chemical messenger b) Speeds nerve impulse transmission c) Carries message to the next nerve cell d) Represents building block of nervous system

c

Heidi Nikolas, a 32-year-old fabric designer, has just received a diagnosis of premature ovarian failure. How would you expect her diagnosis to be confirmed? a) Follicular biopsy b) Menstrual diary c) FSH level determination d) CT scan

b

Hematoma and seroma formation are complications of breast surgery. Which of the following is the indicator that should be reported to the surgeon? a) Bruising of the skin b) Gross swelling c) Tightness of the skin d) Pain at the site

--

How would the nurse prevent rebound hypoglycemia in a patient receiving parenteral nutrition? -Discontinue gradually to allow the patient to adjust to decreased levels of glucose. -If the PN solution is abruptly terminated, isotonic dextrose can be administered at the same rate the PN solution was infusing for 1-2 hours to prevent rebound hypoglycemia. -SXs of rebound hypoglycemia include weakness, faintness, sweating, shakiness, feeling cold, confusion, and increased heart rate.

candidiasis

Infection caused by Candida species or yeast; also referred to monolial vaginitis or yeast infection.

epididymitis

Infection of the epididymus that usually descends from an infected prostate or urinary tract; also may develop as a complication of gonorrhea, chlamydia, or e coli.

mucopurulent cervicitis

Inflammation of the cervix with exudate; almost always related to chlamidyal infection.

salpingitis

Inflammation of the fallopian tubes.

endocervicitis

Inflammation of the mucosa and the glands of the cervix.

vaginitis

Inflammation of the vagina, usually secondary to infection. Group of conditions that cause vulvovaginal symptoms such as itching, irritation, burning, and abnormal discharge. Bacterial vaginitis is the most common, followed by vulvovaginal candidiasis and trichomoniasis. Vaginal discharge becomes more profuse before onset of menstrual cycle when vaginitis is present. Discharge may produce itching, odor, redness, burning, or edema, which may be aggravated by voiding and defecation. Urethritis may accompany vaginitis due to close proximity.

vulvitis

Inflammation of the vulva, usually secondary to infection or irritation.

vestibulitis

Inflammation of the vulvar vestibule.

PID (pelvic inflammatory disease)

Inflammatory condition of the pelvic cavity, usually from an STI.

a

Kara Carpenter is a 54-year-old woman who just had a left radical mastectomy. The nurse caring for her is providing information on complications that may arise due to removing the axillary lymph nodes. Which of the following would not be included? a) All would be included in the discussion. b) Tissue necrosis c) Reduced range of motion d) Infection

hyphae

Long, branching filamentous structure characteristic of fungi such as Candida seen under microscopic examination.

pelvic exenteration

Major surgical procedure in which the pelvic organs are removed.

pancreatitis

Mechanisms causing pancreatic inflammation are unknown, commonly described as autodigestion of the pancreas. It is believed that the pancreatic duct becomes temporarily obstructed, accompanied by hypersecretion of the exocrine enzymes of the pancreas, These enzymes enter the bile duct, where they are activated, and together with bile, back up (reflux) into the pancreatic duct, causing _____.

cryptorchidism

Most common congenital defect in males; characterized by failure of one or both of the testes to descend into the scrotum.

vestibulodynia

Most common type of vulvodynia. Characterized by sharp pain in response to pressure applied to the vestibular area of the vulva.

BPH

Noncancerous enlargement or hypertrophy of the prostate; the most common pathologic condition in older men.

epiglottitis

Nursing Management • Do not leave child unattended • Do not place patient in supine position may cause airway occlusion • Provide 100% oxygen in least invasive manner • Complete airway occlusion emergency tracheostomy • TAKE NOTE - PAGE 610 o Characterized by dysphagia, drooling, anxiety, irritability, & significant respiratory distress. Be prepared for sudden airway occlusion.

prostatism

Obstructive and irritative symptom complex that includes increased frequency and hesitancy in starting urination, a decrease in the volume and force of the urinary stream, acute urinary retention, and recurrent UTIs.

c

On a follow-up visit, the patient is informed that her breast cancer has recurred. The nurse knows which of the following situations places the patient at risk for developing bone metastases? a) No maternal family history of breast cancer b) Aggressive treatment, including surgery, radiation, and hormonal therapy c) Recurrence within 2 years of the original diagnosis d) Previous therapeutic response to chemotherapy

vvulvodynia

Painful condition that affects the vulva. Chronic, unexplained pain in the area around the opening of the vagina. Can't be cured, but treatment may help.

appendicitis

Physical Examination • Appear anorexic & ill • Can't walk or climb on exam table without assistance • Maximal tenderness over McBurney point in the RLQ on palpation • IF PAIN SUDDENLY GOES AWAY-CALL DR-PERFORATION HAS OCCURRED • Assess abdomen for acute peritonitis o Diffuse abdominal tenderness or distention o Call Dr immediately with positive findings

conization

Procedure in which a cone-shaped piece of cervical tissue is removed as a result of detection of abnormal cells; also called cone biopsy.

LEEP (loop electrocautery excision procedure)

Procedure in which a laser is used to remove a thin layer of cervical tissue after an abnormal biopsy finding.

Depolarization

Process by which cardiac muscle cells change from a more negatively charge to a more positively charged intracellular state

spermatogensis

Production of sperm in the testes.

pannus

Proliferation of newly formed synovial tissue infiltrated with inflammatory cells. Seen in RA. Destroys cartilage and erodes bone resulting is loss of articular surfaces and joint motion.

brachytherapy

Radiation delivered by an internal device placed close to the tumor.

salpingo-oophorectomy

Removal of the ovary and its fallopian tube.

vulvectomy

Removal of the tissue of the vulva.

colporrhaphy

Repair of the vagina.

douche

Rinsing the vaginal canal with fluid.

renal

Risk factors for ____ cancer: -Gender - male -Tobacco use -Occupational exposure to industrial chemicals, such as petroleum products, heavy metals, asbestos -Obesity -Unopposed estrogen therapy -Polycystic kidney disease

c

Samantha Velasquez, a 24-year-old preschool teacher, is being seen by the physician in the primary care group where you practice nursing. Over the past 2 months, she has been receiving treatment for multiple ear infections and tonsillitis. She reports a curdy white vaginal discharge and burning with urination. What is the most likely cause of her symptoms? a) Gardnerella vaginalis b) Trichomonas vaginalis c) Candida albicans d) None of the above

b

Sentinel lymph node mapping is done to validate the lack of lymph node metastasis. Which of the following complications does this technique help avoid? a) Breast cancer b) Lymphedema c) Mastalgia d) Fibroadenoma

a

Students are reviewing information about the use of adjuvant hormonal therapy for breast cancer. They demonstrate understanding of this information when they identify which of the following as an example of a selective estrogen receptor modulator (SERM)? a) Tamoxifen b) Anastrozole c) Exemestane d) Letrozole

ADT (androgen deprivation therapy)

Surgical (orchiectomy) or medical castration (e.g., with luteinizing hormone-releasing hormone agonists).

cystostomy

Surgical creation of an opening into the urinary bladder.

oophorectomy

Surgical removal of an ovary.

myomectomy

Surgical removal of uterine fibroids.

perineorrhaphy

Surgical repair of perineal lacerations.

gastric cancer

Symptoms of ____ ____ early disease, such as pain relieved by antacids, resemble those of benign ulcers and are seldom definitive. Symptoms of progressive disease include dyspepsia (indigestion), early satiety, weight loss, abdominal pain just above the umbilicus, loss or decrease in appetite, bloating after meals, N/V, and symptoms similar to those of peptic ulcer disease.

dysplasia

Term related to abnormal cell changes; may be found on Pap smear and cervical biopsy reports.

sq

The caregiver of a patient with Parkinson's disease receives nursing instructions about the administration of apomorphine (Apokyn). What information should the nurse include in the teaching plan for the caregiver? -Administer the medication _____.

c

The client arrives at a public health clinic worried that she has breast cancer since finding a lump in her breast. When assessing the breast, which assessment finding is characteristic of fibrocystic disease? a) The lump is firm and nonmovable. b) Nipple retractions are noted. c) The lump is round and movable. d) One breast is larger than the other.

b

The client is asking if there is a pill that can be ordered to control the symptoms of menopause. Which assessment finding is most important in determining nursing care in association with hormone replacement therapy? a) Presence of kyphosis b) Family history of breast cancer c) History of osteoporosis d) Symptoms of hot flashes

d

The client returns to the nursing unit following an open reduction with internal fixation of the right hip. Nursing assessment findings include temperature 100.8 degrees Farenheit, heart rate 112 beats per minute, respiratory rate 28 breaths per minute, and blood pressure 86/58. There is no urine in the Foley catheter collection bag. The nurse interprets these findings as indicating which of the following complications? a) Osteomyelitis b) Urinary retention c) Atelectasis d) Hypovolemic shock

b (Manifestations of a dislocated hip prosthesis are increased pain at the surgical site, acute groin pain in the affected leg, shortening of the leg, abnormal external or internal rotation, and limited movement of the fractured leg. The client may report a "popping" sensation in the hip)

The client who had a total hip replacement was discharged home and developed acute groin pain in the affected leg, shortening of the leg, and limited movement of the fractured leg. The nurse interprets these findings as indicating which of the following complications? a) Re-fracture of the hip b) Dislocation of the hip c) Contracture of the hip d) Avascular necrosis of the hip

a

The diagnosis of multiple sclerosis is based on which of the following tests? a) Magnetic resonance imaging (MRI) b) Neuropsychological testing c) Evoked potential studies d) Cerebrospinal fluid (CSF) electrophoresis

respiratory

The early manifestations of amyotrophic lateral sclerosis (ALS) and MS are somewhat similar. Which clinical feature of ALS distinguishes it from MS? Impairment of _____ muscles.

d (If any clinical manifestations of dislocation of the prosthesis occur, including acute groin pain in the affected hip or shortening of the affected extremity, the nurse must immediately notify the surgeon, because the hip must be reduced and stabilized promptly so that the leg does not sustain circulatory and nerve damage)

The nurse assesses a patient after total right hip arthroplasty and observes a shortening of the extremity, and the patient complains of severe pain in the right side of the groin. What is the priority action of the nurse? a) Apply Buck's traction. b) Externally rotate the extremity. c) Bend the knee and rotate the knee internally. d) Notify the physician.

sulfa

The nurse is assessing a patient who is going to begin therapy with apomorphine for Parkinson's disease. Which condition would be a concern if identified in the patient? -Severe cardio vascular disease -Allergy to _____ drugs

d

The nurse is assessing an older adult female who has not seen her physician in 2 years. The nurse is assisting the patient into a gown and notices that the patient has edema and pitting of the skin on the right breast. What does the nurse understand is the significance of this finding? a) This finding is not uncommon and is significant only when of recent origin. b) It may result from inflammation due to mastitis while the patient is breastfeeding. c) This finding is most likely related to benign cysts of the breast in the nipple area. d) It may result from a neoplasm blocking lymphatic drainage, giving the skin an orange-peel appearance, a classic sign of advanced breast cancer.

b

The nurse is assessing the breast of a female patient and observes a prominent venous pattern on the left breast. What does the nurse understand that this can be indicative of? a) Ulceration of the nipple b) Increased blood supply required by a tumor c) Infection d) Thrombus formation

d

The nurse is caring for a client who is beginning doxorubicin (Adriamycin) therapy for breast cancer. When preparing the client for probable side effects, which would the nurse include? a) Information regarding high caloric meals from a dietician b) Information regarding depression from a mental health association c) Information about blood donation from the American Red Cross d) Information regarding wigs from the American Cancer Society

c

The nurse is caring for a group of breast cancer survivors post mastectomy. When developing a list of instructions of points to avoid, which point is highlighted? a) Arm exercises on affected side b) Applying cream to breast c) No lifting greater than 15 lb d) Wearing loose fitting shirts

a

The nurse is caring for a patient who had a total hip replacement. What lethal postoperative complication should the nurse closely monitor for? a) Pulmonary embolism b) Urinary tract infection c) Hypovolemia d) Atelectasis

b

The nurse is caring for patient with a hip fracture. The physician orders the patient to start on a bisphosphonate. Which medication would the nurse document as given to treat or prevent osteoporosis? a) Raloxifene (Evista) b) Alendronate (Fosamax) c) Denosumab (Prolia) d) Teriparatide (Forteo)

B

The nurse is encouraging a patient to have a cervical examination and Pap smear. It has been many years since the patient's last exam, and she was diagnosed with HPV 6 years ago. The patient states, "I am not having any trouble down there, so it is best to leave things alone." What is the best response by the nurse? a) "If you are not having any problems, then there is no reason to have one." b) "Early cervical cancer rarely produces any symptoms." c) "You could have another type of sexually transmitted infection." d) "If your insurance is paying for it, you should have an exam."

b

The nurse is performing an initial assessment on a client admitted with a possible brain abscess. Which of the following would the nurse most likely find? a) Nuchal rigidity b) Headache that is worse in the morning c) Diplopia that is constant d) Ptosis that is more pronounced at the end of the day

B

The nurse is preparing a discharge teaching plan for a client who has had a prostatectomy. Which of the following would be appropriate to include? a) Waiting to urinate for 5 to 10 minutes after feeling the initial urge b) Performing perineal exercises frequently throughout the day c) Engaging in strenuous exercise to strengthen abdominal muscles d) Using a bearing down motion to promote complete bladder emptying when voiding

C

The nurse is preparing a presentation for a men's community group about health promotion. Which of the following would the nurse include as a current recommendation for screening? a) Transrectal ultrasound every 5 years after age 50 years b) Annual digital rectal examination (DRE) after age 35 years c) Monthly testicular self-examination (TSE) d) Annual prostate-specific antigen (PSA) testing after age 40 years

a

The nurse is preparing a teaching plan for a client with a vulvovaginal infection. Which of the following would be least appropriate for the nurse to include? a) Performing douching with a dilute vinegar solution twice a day b) Maintaining a reclining position for 30 minutes after inserting vaginal medication c) Refraining from unprotected sexual intercourse with partners d) Wearing cotton underwear that is loose-fitting and allows for air flow

a

The nurse is providing care to a client who has had surgery as treatment for breast cancer. The nurse would be alert for the development of which of the following? a) Lymphedema b) Fibrocystic breast disease c) Breast abscess d) Fibroadenoma

D

The nurse is providing instruction for testicular self-examination to a group of young adolescents. Which is the most correct examination technique? a) Palpate both testicles simultaneously for comparison. b) Palpate for a soft, round shape with normal ridges on the testicles. c) Palpate the front of the testicle first, where most tumors are found. d) Palpate each testicle separately, following a warm shower.

d (An antecedent event, most often a viral infection, precipitates clinical presentation. The antecedent event usually occurs about 2 weeks before the symptoms begin)

The nurse is taking health history from a client admitted to rule out Guillain-Barre syndrome. An important question to ask related to the diagnosis is which of the following? a) "Have you developed any new allergies in the last year?" b) "Have you experienced any ptosis in the last few weeks?" c) "Have you had difficulty with urination in the last 6 weeks?" d) "Have you experienced any viral infections in the last month?"

b (may cause hip dislocation)

The nurse teaches the patient which of the following interventions in order to avoid hip dislocation after replacement surgery? a) Keep the knees together at all times b) Never cross the affected leg when seated c) Avoid placing a pillow between the legs when sleeping d) Bend forward only when seated in a chair


Conjuntos de estudio relacionados

Leadership & Management - UWorld

View Set

Combo with "TAM 1200 FINAL EXAM" and 11 others

View Set

Chapter 41: Gastrointestinal Dysfunction

View Set

Chapter Exam - Life Underwriting

View Set

Chapter 22, Section 1 : Life in the industrial Age

View Set

CLEP Human Growth and Development

View Set

Cell and Molecular Biology, Chapter 1

View Set

Cog Psych Chapter 10, Cog Psych Chapter 11, Chapter 12, Cog Psych Chapter 13

View Set

Bldg Const. Chapter 13: Concerns of Green Construction

View Set

Post WW2 History and Society (Praxis 5004)

View Set

ECON 230D2 Ch 13 - Oligopoly & Monopolistic Competition

View Set